Question Bank 2015-2017 Flashcards

You may prefer our related Brainscape-certified flashcards:
1
Q

An Examiner you have dealt with for many years calls you today to advise you as a courtesy that you filed, apparently by mistake, a letter from your client discussing in detail the competitive situation in his industry and asking you to keep this information in strictest confidence. You call the client who makes extreme threats about taking everything you have now or will ever have if this letter becomes public. You should:

(A) Call Tony Stark and see if he has any ideas for getting the letter back.
(B) File a petition and pay the required fee to expunge the letter and return it to you.
(C) Start spending whatever you have because you won’t have it for long.
(D) Ask your friend the Examiner to just mail it back to you or tear it up.
(E) Tell your client the letter has been destroyed and pray he never finds out to the contrary.

A

(B) is correct. See 37 C.F.R. 1.59. Information forming part of the original disclosure (i.e., written specification including the claims, drawings, and any preliminary amendment present on the filing date of the application) will not be expunged from the application file. However, an applicant may request that the Office expunge information (other than the original disclosure) by filing a petition. Any petition to expunge information from an application must include the fee and establish to the satisfaction of the Director that the expungement of the information is appropriate.

(A) is incorrect, although asking Iron Man for assistance has to come in a close second.
(C) is unnecessary because you can fix the problem.
(D) is not something a patent examiner can, or should, do.
(E) is incorrect because it would be unethical. Never pick the unethical answer on this Exam.

How well did you know this?
1
Not at all
2
3
4
5
Perfectly
2
Q

Your first job after passing the exam is with the largest and best patent law firm in the United States. You were able to lease both a Mercedes and Jeep out of your first month’s pay. Next month you start repaying that student loan. Your new boss returns the amendment you prepared together with a memo saying that he has discussed the amendment with the client and the client believes he is entitled to a broad claim that reads on the prior art. Your boss tells you to submit the amendment with such a broad claim ending the memo:

“I agree that such a claim is not patentable, but let’s let the Examiner tell this guy. He has a history of firing attorneys who won’t do what he wants.” Rule 11.18:

(A) Allows you to ethically file the amendment with the broad claim, because your client has a bona fide, but mistaken view that the claim is patentable.
(B) Forbids submitting such a claim.
(C) Allows the claim to be submitted so long as the client signs the amendment.
(D) Applies only to practitioners, not to other members of the public.
(E) Has no mechanism for enforcing its requirements.

A

(B) is correct. See 37 C.F.R. 11.18. This question requires you to suspend belief a bit; not because you get a great job and are making lots of money, but because you must stand up to your boss and tell them that Rule 11.18 forbids asking for a claim you know you are not entitled to receive. NOTE: Do not fight the facts. In life, there may be ways to finesse any set of claims or likely rejection. Here, you are clearly being told to assume that the claim is not patent eligible. In this situation, there is only one right answer - do not ask for a claim you know you are not entitled to receive.

How well did you know this?
1
Not at all
2
3
4
5
Perfectly
3
Q

The America Invents Act amends U.S. patent laws, among other things, to do which of the following?

(A) Convert the U.S. patent system from a “first to invent” system to a “first inventor to file” system.
(B) Treat U.S. patents and U.S. patent application publications as prior art as of their earliest effective filing date, regardless of whether the earliest effective filing date is based upon an application filed in the United States or in another country.
(C) Eliminate the requirement that a prior public use or sale be “in this country” to be a prior art activity.
(D) Treat commonly owned or joint-research-agreement patents and patent application publications as being by the same inventive entity for purposes of 35 U.S.C. 102, as well as 35 U.S.C. 103.
(E) All of the above.

A

(E) is correct. All of the above are true statements. Section 3 of the AIA, among other things, amends the patent laws to: (1) convert the U.S. patent system from a “first to invent” system to a “first inventor to file” system; (2) treat U.S. patents and U.S. patent application publications as prior art as of their earliest effective filing date, regardless of whether the earliest effective filing date is based upon an application filed in the United States or in another country; (3) eliminate the requirement that a prior public use or sale be “in this country” to be a prior art activity; and (4) treat commonly owned or joint research agreement patents and patent application publications as being by the same inventive entity for purposes of35 U.S.C. 102, as well as 35 U.S.C. 103. These changes in section 3 of the AIA are effective on March 16,2013, but apply only to certain applications filed on or after March 16,2013.

How well did you know this?
1
Not at all
2
3
4
5
Perfectly
4
Q

Enya Brennan files a patent application directed to an improved music machine in Ireland on March 17, 2013. She subsequently files an international patent application under the PCT in Ireland on May 17, 2013, claiming the benefit of the earlier Irish filing. Upon learning of the ability to accelerate applications out of turn in the U.S., she files a nonprovisional patent application in the U.S. on July 4,2013, which claims the benefit of both the Irish filing and PCT filing, together with a request for Track One acceleration and payment of the appropriate fee. She receives a Notice of Allowance on October 15, 2013, and a U.S. patent issues December 31,2013.

What is the earliest date the Enya disclosure as filed will be considered prior art for subsequent applicants who file in the U.S.?

(A) March 17,2013
(B) May 17,2013
(C) July 4, 2013
(D) October 15, 2013
(E) December 31, 2013

A

(A) is correct. The AIA eliminates the so-called Hilmer doctrine. Under the Hilmer doctrine, pre-AIA 35 U.S.C. 102(E) limited the effective filing date for U.S. patents (and published applications) as prior art to their earliest U.S. filing date. In contrast, AIA 35 U.S.C. 102(D) provides that if the U.S. patent, U.S. patent application publication, or WIPOpublished application claims priority to one or more prior-filed foreign or international applications under 35 U.S.C. 119 or 365, the patent or published application was effectively filed on the filing date of the earliest such application that describes the subject matter. Therefore, if the subject matter relied upon is described in the application to which there is a priority or benefit claim, a U.S. patent, a U.S. patent application publication, or WIPO-published application is effective as prior art as of the filing date of the earliest such application, regardless of where filed.

Thus, the Irish filing date of March 17,2013, will be the earliest date that the Enya disclosure will be considered prior art against subsequent applications.

How well did you know this?
1
Not at all
2
3
4
5
Perfectly
5
Q

Keith Urbayne files a patent application directed to a self-preservation device to be used when caught between two divas who don’t very much like each other. He files a patent application in Australia on April 15, 2013. Subsequently, Urbayne files an international patent application under the PCT in Australia on June 1, 2013, claiming the benefit of the earlier Australian filing. The Australian patent application publishes on October 15, 2013, and the PCT application publishes December 1, 2013. Having soured on the U.S. thanks to an unfortunate reality-TV experience, Urbayne does not enter the U.S. national stage or otherwise seek a U.S. patent.

What is the earliest date the Urbayne disclosure as filed will be considered prior art for subsequent applicants who file in the U.S.?

(A) April 15, 2013
(B) June 1, 2013
(C) October 15, 2013
(D) December 1, 2013
(E) It will not be prior art in the U.S. because no U.S. filing was ever made

A

(A) is correct. The AIA eliminates the so-called Hilmer doctrine. Under the Hilmer doctrine, pre-AIA 35 U.S.C. 102(E) limited the effective filing date for U.S. patents (and published applications) as prior art to their earliest U.S. filing date. In contrast, AIA 35 U.S.C. 102(D) provides that if the U.S. patent, U.S. patent application publication, or WIPO published application claims priority to one or more prior-filed foreign or international applications under 35 U.S.C. 119 or 365, the patent or published application was effectively filed on the filing date of the earliest such application that describes the subject matter. Therefore, if the subject matter relied upon is described in the application to which there is a priority or benefit claim, a U.S. patent, a U.S. patent application publication, or WIPO-published application is effective as prior art as of the filing date of the earliest such application, regardless of where filed. This question is very similar to the previous question dealing with Enya. The only difference here is that Urbayne did not ultimately seek or obtain a U.S. patent. That does not matter, however. Because there was a publication of the PCT application, the Australian filing date of April 15, 2013, will be the earliest date that the Urbayne disclosure will be considered prior art against subsequent applications.

How well did you know this?
1
Not at all
2
3
4
5
Perfectly
6
Q

During the course of a long trial for patent infringement in Los Angeles, testimony from an expert during the day convinces you that you should file a continuation of a pending design application. That application has an outstanding Official Action, which is dated exactly six months ago. No response has been filed and you intend to allow the application to become abandoned. It is now 9:30 PM in Los Angeles and today is a Tuesday in August. Your best course of action would be:

(A) Transmit a request by facsimile to file a continuation under Rule 1.53(D) to the Patent Office including a request for a three-month extension of time.
(B) Transmit a request by facsimile to file a continuation under Rule 1.53(D) to the Patent Office including a request for a three-month extension of time, and a Certificate of Transmission.
(C) File a Rule 1.53(B) application tomorrow together with a four-month extension of time.
(D) File a continuation under Rule 1.53(D) via EFS including a request for a three month extension of time.
(E) Find a Post Office that is open and transmit a Rule 1.53(B) or (D) application to the Patent Office by Express Mail before Midnight together with a request for a three month extension of time.

A

(E) is correct. 37 CFR 1.6. Correspondence transmitted by facsimile to the Patent and Trademark Office will be stamped with the date on which the complete transmission is received in the Patent and Trademark Office unless that date is a Saturday, Sunday, or Federal holiday within the District of Columbia, in which case the date stamped will be the next succeeding day which is not a Saturday, Sunday, or Federal holiday within the District of Columbia. Since it is already 9:30pm in Los Angeles that means it is 12:30am in Alexandria, Virginia, where the Patent Office is located. Thus, it is already the day after the last day to take action at the USPTO, so filing a CPA via fax (which is otherwise possible) cannot be accomplished because it will not be given a filing date as of the time filed in the time zone where you are located. The only way to get the benefit of the time zone where you are located is to file the old-fashioned way, which is to file via U.S. Postal Service Express Mail. NOTE: You must pay a 3-month extension fee because you would have been given a 3-month shortened statutory period to respond and since it is the last day of the 6-month statutory period, you have to pay for 3 months of extension to do anything, including filing a continuation that claims the benefit of that pending application.
(A) and (B) are incorrect because, even with a Transmission Certificate, the time of receipt governs and the CPA will get a filing date that is the day after the 6-month statutory period expired. The same applies to (D) as well. EFS is governed by date and time received at the USPTO.

(C) is incorrect because there is no such thing as a 4-month extension to respond to an Office Action. This choice puts you beyond the 6-month statutory period to respond.

How well did you know this?
1
Not at all
2
3
4
5
Perfectly
7
Q

Which of the following is true?

(A) A statement with respect to a translation being submitted in response to a Notice of Missing Parts must state that the translation is accurate and that the translator is fluent in both English and the language being translated.
(B) If the inventors identified in a nonprovisional application which is filed without an oath or declaration differs from the inventors who execute the oath or declaration subsequently filed, the inventorship must be corrected under Rule 1.48.
(C) A provisional application (claiming foreign priority from a U.K. application) can be filed under Rule 1.53(C).
(D) A five-month automatic extension of time to file an appeal brief in an appeal of a utility application is always possible.
(E) A newly executed oath must always be submitted in filing an application under Rule 1.53(B).

A

(D) is correct. The 6-month statutory deadline does not apply with respect to filing an appeal brief. Similarly, the 6-month statutory deadline does not apply to responding to a Notice of Missing Parts. In these two situations, you will be given 2 months to respond and they are fully extendable, meaning 5 months of extension are available. These two peculiar situations are the only two that allow you to go beyond the 6-month statutory deadline, so you should be prepared to see them on the exam.

(A) is incorrect because no statement of competence is required.

(B) is incorrect because only the inventors named in the oath or declaration are official.

(C) is incorrect because a provisional application cannot claim priority.

(E) is incorrect because a copy of the original oath can be submitted in filing a continuation under Rule 1.53(B). Notice, however, that if you are filing a continuation-in-part, a new oath or declaration would be warranted. A continuation contains no new matter not in the parent application. A continuation-in-part does contain matter not in the parent application; thus, the parent oath/declaration would not cover that newly added information in a continuation-in-part.

How well did you know this?
1
Not at all
2
3
4
5
Perfectly
8
Q

Your client Jenny Smith comes to you with happy news. IBM has taken a nonexclusive field of use license under her pending application, which has an issue fee due. No money was received and she is unsure when payment might be received, if ever. At the time the application was filed you filed a Small Entity Declaration. You should now:
(A) Notify the Patent Office of the change in status.
(B) Repay the amounts saved in filing and prosecuting the application as a Small Entity.
(C) Do nothing because Small Entity status is not lost by the grant of a non-exclusive license.
(D) Pay the issue fee as a Small Entity and consider repayment when money is received from IBM.
(E) Ask IBM to pay the issue fee.

A

(A) is correct. 37 CFR 1.27. A small entity is one that has not assigned, granted, conveyed, or licensed, and is under no obligation under contract or law to assign, grant, convey, or license, any rights in the invention. An inventor or other individual who has transferred some rights in the invention to one or more parties, or is under an obligation to transfer some rights in the invention to one or more parties, can also qualify for small entity status if all the parties who have had rights in the invention transferred to them also qualify for small entity status. Thus, while Jenny is a small entity, IBM is not a small entity. So, she will have to notify the Patent Office of the change in status. Note, however, that once status as a small entity has been established in an application or patent, fees as a small entity may thereafter be paid in that application or patent without regard to a change in status until the issue fee is due or any maintenance fee is due.

(B) is incorrect because you do not have an obligation to go backwards and pay any deficiency.

(C) is incorrect because a license, even a nonexclusive license, does cause a change in small entity status.

(D) is incorrect because you are required to notify the Patent Office of the change and pay the appropriate fees.
(E) is something you may be able to try and negotiate, but it doesn’t change the fact that you must do
(A).

How well did you know this?
1
Not at all
2
3
4
5
Perfectly
9
Q

Your client Wilson Blue calls you so angry he can hardly speak. He has learned that his bitterest competitor and ex-spouse has visited each of his customers, told them she has an as-yet-unpublished patent application issuing in two months which they will infringe and demanding that they stop selling Blue’s product. In fact, she repeatedly told them that she will ruin them in such an infringement case. All but one has dropped your client. You should advise Blue:
(A) He has insufficient standing because as yet she does not have an issued patent; therefore she cannot yet bring a lawsuit.
(B) There is nothing to worry about. What she is telling Blue’s clients cannot be correct since you have three months, not two months, to pay the issue fee.
(C) Petition the Commissioner for access to her unpublished application on the grounds of Special Circumstances.
(D) Since he is no longer married to his ex-spouse, there is no way to access her application on file at the USPTO.
(E) You can access the application since the issue fee has apparently been paid. 15.

A

(C) is correct. See 35 USC 122 and 37 CFR 1.14(i). A petition to access an otherwise confidential application must: (1) include payment of the fee; and (2) include a showing that access to the application is necessary to carry out an Act of Congress or that special circumstances exist which warrant petitioner being granted access to all or part of the application. The situation described is a situation where there are special circumstances.

How well did you know this?
1
Not at all
2
3
4
5
Perfectly
10
Q

Which of the following can you do as a matter of right in a non provisional patent application?

(A) File a substitute specification if accompanied by a statement of no new matter and a marked up copy.
(B) Conduct an interview after a Final Rejection.
(C) Appeal any Official Action that rejects the claims.
(D) Submit an IDS after a First Official Action upon a showing that it could not have been submitted earlier.
(E) Withdraw from representing a client at any time before the six-month deadline for responding to an Official Action.

A

(A) is correct. See 37 CFR 1.125. A substitute specification, excluding the claims, may be filed at any point up to payment of the issue fee if it is accompanied by a statement that the substitute specification includes no new matter. However, a substitute specification is not permitted in a reissue application or in a reexamination proceeding.

How well did you know this?
1
Not at all
2
3
4
5
Perfectly
11
Q

AIA 35 U.S.C. 102(B)(2)(C) provides that a disclosure made in a U.S. patent, U.S. patent application publication, or WIPO-published application is not prior art to a subsequent claimed invention under 35 U.S.C. 102(A)(2) if, not later than the _________ - ‘’the subject matter disclosed and the claimed invention were owned by the same person or subject to an obligation of assignment to the same person.”

(A) effective filing date of the subsequent claimed invention …
(B) effective filing date of the first invention …
(C) date of conception of the first invention …
(D) date of conception of the subsequent invention …
(E) None of the above.

A

(A) is correct. AIA 35 U.S.C. 102(B)(2)(C) provides that a disclosure made in a U.S. patent, U.S. patent application publication, or WIPOpublished application is not prior art to a claimed invention under 35 U.S.c. 102(A)(2) if, not later than the effective filing date of the claimed invention, the subject matter disclosed and the claimed invention were owned by the same person or subject to an obligation of assignment to the same person. This provision replaces the exception in pre-AIA 35 U.S.C. 103(C) that applied only in the context of an obviousness analysis under 35 U.S.C. 103 to prior art that was commonly owned at the time the claimed invention was made. See 35 U.S.c. 102(B)(2)(C).

How well did you know this?
1
Not at all
2
3
4
5
Perfectly
12
Q

With respect to subject matter made pursuant to joint research agreements, which of the following are required to demonstrate the claimed invention was owned by the same person or subject to an obligation of assignment to the same person?

I. The subject matter disclosed was developed and the claimed invention was made by, or on behalf of, one or more parties to a joint research agreement that was in effect on or before the effective filing date of the claimed invention.
II. The claimed invention was made as a result of activities undertaken within the scope of the joint research agreement.
III. The application for patent for the claimed invention discloses or is amended to disclose the names of the parties to the joint research agreement.

(A) I
(B) II
(C) I and II
(D) I and III
(E) I, II and III

A

(E) is correct. Each of II and III are correct. Under 35 U.S.C. 100(h), the term “joint research agreement” as used in AIA 35 U.S.C. 102(C) is defined as a written contract, grant, or cooperative agreement entered into by two or more persons or entities for the performance of experimental, developmental, or research work in the field of the claimed invention. AIA 35 U.S.C. 102(C) specifically provides that subject matter disclosed and a claimed invention is deemed to have been owned by the same person or subject to an obligation of assignment to the same person in applying the provisions of AIA 35 U.S.C. 102(B)(2)(C) if: (1) the subject matter disclosed was developed and the claimed invention was made by, or on behalf of, one or more parties to a joint research agreement that was in effect on or before the effective filing date of the claimed invention; (2) the claimed invention was made as a result of activities undertaken within the scope of the joint research agreement; and (3) the application for patent for the claimed invention discloses or is amended to disclose the names of the parties to the joint research agreement. See AIA 35 U.S.C. 102(B)(2)(C).

How well did you know this?
1
Not at all
2
3
4
5
Perfectly
13
Q

Which of the following statements is not true?

(A) The one-month limit to complete a response found incomplete can be extended.
(B) An application can be accepted for filing and given a filing date even though the inventors are not identified.
(C) Claims, which are cancelled by a Preliminary Amendment filed with the application, are not counted in determining the additional claim feels).
(D) The substance of telephone discussions with an examiner must be made of record in the application file.
(E) A double patenting rejection can be made only if the two applications have at least one inventor in common.

A

(E) is correct. Substantively, a double patenting rejection can be made when two applications have either a common owner or at least one common inventor. Thus, it is incorrect to say such a rejection can only be made when there is at least one common inventor.

How well did you know this?
1
Not at all
2
3
4
5
Perfectly
14
Q

Your client John Wells has at last received the Notice of Allowance covering his new Mexican jumping bean. Mr. Wells is a resident of Costa Rica, and tells you he is worried that someone will start some proceedings in the Patent Office or in some court affecting his patent and he will not be aware because postal delivery is so bad in his country. He wants his brother who lives in Los Angeles to take care of any and all business relating to the patent. You should:
(A) Advise him not to worry because the examiner is a friend of yours and will call you if anything happens.
(B) Prepare for Mr. Wells’ signature a written designation of his brother as the one on whom service should be made concerning the patent.
(C) Advise him that all papers from the Patent Office will be sent to you as attorney of record and you will forward them to him by a secure route.
(D) Prepare and file a Notice of Foreign Patent Owner requesting that all papers be sent to Mr. Wells care of the American Embassy in Costa Rica.
(E) Prepare and file an assignment of the patent to you together with an agreement that you will forward any and all monies received with respect to the patent to Mr. Wells.

A

(B) is correct. See 37 C.F.R. 1.33(A). The Patent Office will correspond with one person at one address. This address is designated the Correspondence Address. (A) is incorrect. The examiner will not keep you posted. The Patent Office communicates officially only with the Correspondence Address. (C) is incorrect. This is what frequently happens but that’s only because the attorney’s address is usually listed as the Correspondence Address. (D) is inferior. While you could list the Embassy as the Correspondence Address, that is not what the client is asking you to do. You can accomplish exactly what the client wants with (B), so (D) is clearly an inferior choice. (E) is incorrect. Having the client sign over the patent to you is full of Ethical land mines. It is unnecessary and likely unethical unless done exactly properly. It also isn’t what the client wants. You can accomplish exactly what the client wants with (B).

How well did you know this?
1
Not at all
2
3
4
5
Perfectly
15
Q

Bernard is an independent inventor who, like so many other independent inventors, does not have the funds necessary to seek advice from a patent attorney. He joined a local inventors group and has learned from some of the more experienced members that the new first-to-file law continues to include a grace period. In fact, he understands that the grace period allows an inventor such as himself to publish a disclosure of his invention and then subsequently file a patent application within 12 months. If you had the opportunity to speak with Bernard, which of the following would you NOT tell him?

(A) The disclosure of the invention in a patent application should be as complete as possible.
(B) Any subsequent disclosure by another will be unable to be used as prior art as long a U.S. patent application is filed within 12 months.
(C) An independently arrived at disclosure by another can be considered prior art and could prevent the issuance of a patent.
(D) If the disclosure describes elements A, B and C, and a subsequent disclosure prior to filing discloses A, B, C and D, then at least element D would be considered prior art.
(E) All of the above.

A

(B) is correct. Many subsequent disclosures will be prior art. There is only a very limited exception available to the inventor under AIA 102(B). Thus, it is incorrect to say that “any subsequent disclosure by another” can be overcome.

(A) is incorrect because a complete disclosure at the time of filing is always good advice since priority attaches only to what arrives at the time of filing.

(C) is incorrect because independent disclosures can be prior art that cannot be overcome, depending on the circumstances.

(D) is incorrect because it is a true statement. The Patent Office gave this example when they released their interpretations of the AIA 102(B) exception: If inventor discloses XY and someone subsequently independently discloses XYZ, at least Z will be prior art against the inventor.

(E) is incorrect because (B) is correct.

How well did you know this?
1
Not at all
2
3
4
5
Perfectly
16
Q

With respect to AIA 102, which of the following is true?

(A) The availability of a disclosure as prior art may be measured from the effective filing date of the claimed invention, no matter where that filing occurred.
(B) The AIA adopts a global view of prior art disclosures and thus does not require that a public use or sale activity be “in this country” to be a prior art activity.
(E) The catch-all “otherwise available to the public” means, among other things, that secret prior sales are not considered prior art.
(D) The patent or application is considered to have been effectively filed, with respect to any subject matter described in the patent or application, as of the earliest of (1) the actual filing date of the patent or the application for the patent containing the claimed invention; or (2) the filing date of the earliest application for which the patent or application is entitled, as to such invention, to a right of priority or the benefit of an earlier filing date.
(E) All of the above.

A

(E) is correct. All of the above are correct.
(A) is correct. Under the AIA, it is possible for even foreign filing dates to be prior art dates. A foreign filing date will be a prior art date if a US patent, US patent application or WIPO publication of a PCT application claims priority to a foreign filing date. This is a big change from preAIA, where foreign filing dates were never prior art events.
(B) is correct. Sales and public use are prior art events under AIA 102 regardless of whether they occur inside or outside the U.S.
(C) is correct. AIA 35 U.S.C. 102(A)(1) contains the additional residual clause “or otherwise available to the public.” Residual clauses such as “or otherwise” or “or other” are generally viewed as modifying the preceding phrase or phrases. Therefore, the Office views the “or otherwise available to the public” residual clause of the AIA’s 35 U.S.C. 102(A)(1) as indicating that secret sale or use activity does not qualify as prior art.
(D) is correct. AIA 35 U.S.C. 102(D) provides that, for purposes of determining whether a patent or application for patent is prior art to a claimed invention under AIA 35 U.S.C. 102(A)(2), the patent or application must be considered to have been effectively filed, with respect to any subject matter described in the patent or application, as of the earlier of the actual filing date of the patent or the application for patent, or the filing date of the earliest application that describes the subject matter and for which the patent or application for patent is entitled to claim a benefit or right of priority under 35 U.S.C. 119, 120, 121, or 365.

How well did you know this?
1
Not at all
2
3
4
5
Perfectly
17
Q

Your client GS Inc. is the owner of an application that is now allowed as a result of your skill in drafting claims and prosecuting applications. You are instructed to pay the issue fee and you transmit the properly filled out form to the Patent Office the day before the three-month deadline together with your check. Unfortunately due to your bad eyesight you send a check that is $1 less than the amount due. Your letter accompanying the payment broadly authorizes the Patent Office to charge any additional fees that may be due to your deposit account, which contains ample funds. The result will be:
(A) The application will become abandoned because a general authorization is not sufficient to allow the Patent Office to debit your account.
(B) The application will become abandoned because deposit accounts were abolished in 2005.
(C) The Patent Office will send you an incomplete issue fee notice setting a one-month period to pay the deficiency.
(D) The Patent Office will call you to advise you of your error.
(E) None of the above.

A

(E) is correct. See 37 C.F.R. 1.311(B) and 37 C.F.R. 1.136
(A)(3). An authorization to charge the issue fee to a deposit account may be filed in an individual application only after mailing of the notice of allowance. A general authorization on file prior to the sending of the notice of allowance will not operate to pay the issue fee. The USPTO is offering a patent with a notice of allowance. Some action must be taken to accept that offer of a patent. Here, there was appropriate action taken, albeit payment of less than the issue fee together with a specific authorization to look to the deposit account for the additional necessary funds. This suffices and the USPTO will seek the additional $1 from the attorney’s deposit account. (A) is incorrect. It is true that a general authorization is not sufficient, but paying the incorrect issue fee does allow the USPTO to look to the deposit account to correct the deficiency. See 37 CFR 1.311 (b )(1).

(B) is incorrect because deposit accounts were not abolished.
(C) and (D) are incorrect because the Office will merely take the $1 from the attorney’s deposit account.

How well did you know this?
1
Not at all
2
3
4
5
Perfectly
18
Q

Arnold Horshack is the inventor of a new and improved turkey call, which he conceived and reduced to practice on January 30, 2012. The device creates the most obnoxious sound, which approximates a wheezing hyena. Field studies show that turkeys appear to be drawn to the noise out of curiosity, or perhaps pity for what sounds like a dying animal. Horshack files a patent application in Canada on April 1, 2012, and on May 1, 2012, he files a U.S. patent application claiming priority to his Canadian filing date. For financial reasons, the U.S. application is expressly abandoned on July 1, 2012. The Canadian application publishes on October 1, 2013.

Freddie Washington files a U.S. non provisional patent application on November 1, 2013, which relates to a turkey call that approximates a wheezing coyote.

Which of the following would be the earliest date the Horshack disclosure could be used as prior art against Washington?

(A) January 12, 2012.
(B) April 1, 2012.
(C) May 1, 2012.
(D) July 1, 2012.
(E) October 1, 2013.

A

(E) is correct. U.S. patents and U.S. patent application publications can be prior art as of their earliest effective filing date, regardless of whether the earliest filing date is based upon an application filed in the U.S. or in another country. For example, AIA 35 U.S.C. 102(D) provides that if the U.S. patent, U.S. patent application publication, or WIPO-published application claims priority to one or more prior-filed foreign or international applications under 35 U.S.C. 119 or 365, the patent or published application was effectively filed on the filing date of the earliest such application that describes the subject matter. Thus, a foreign filing date can be a prior art date. But will we have a published U.S. application, an issued U.S. patent or a published PCT? There is no mention of a PCT application, and the U.S. application was abandoned so there will be no U.S. patent. Thus, this question turns on whether there will be a U.S. published application that claims priority to Horshack’s Canadian filing date. A U.S. patent application will be published promptly after the expiration of a period of 18 months from the earliest filing date for which a benefit is sought under title 35, United States Code, unless the application is no longer pending. See 37 C.F.R. 1.211(A)(1). Because the Horshack U.S. patent application was expressly abandoned, it will not publish. Thus, Horshack will not obtain a U.S. patent and there will be no U.S. publication. Therefore, the Canadian filing date will not be the prior art date. That means the earliest prior art date would be the date of the Canadian publication, making
(E) the correct answer.

How well did you know this?
1
Not at all
2
3
4
5
Perfectly
19
Q

Jasper Collins filed a patent application on February 15, 2012. He received a Notice of Allowance on January 7, 2014, paid the issue fee on February 5, 2014, and the patent issued April 15, 2014. During the prosecution of his original patent application, he made a substantial improvement. He filed a continuation-in-part application on January 10, 2014, which included disclosure not previously filed and claims drawn to that disclosure, as well as other claims not previously presented but which were previously supported. Because this new improvement was so important and there were significant licensing opportunities, Jasper paid the fee to accelerate the CIP. In the First Office Action on the Merits, which was dated February 28, 2014, the examiner rejected Jasper’s claims based on prior art under AIA 35 U.S.C. 102(A)(1). Surprised that the examiner applied AIA 35 U.S.C. 102(A)(1), Jasper comes to you for help on March 3, 2014. Which of the following represents the best advise to give Jasper?

(A) Because the CIP contained claims not supported until May 16, 2013, the entire CIP will be treated under AIA.
(B) Because the issue fee has been paid in the previous nonprovisional application, it is too late to do anything.
(C) Because the Notice of Allowance has already been received, it is too late to do anything.
(D) The CIP will be treated under AIA, but since the parent application has not yet issued and remains pending a continuation of the parent could be filed and the claims having support dating back to the parent application will then be treated under pre-AIA.
(E) Cancel the claims in the CIP that are directed to the matter not presented until the January 10, 2014, filing.

A

(D) is correct. If an application on filing contains at least one claim having an effective filing date before March 16,2013, and at least one claim having an effective filing date on or after March 16, 2013, the application will be examined under AIA even if the latter claims are cancelled.

Thus (E) is incorrect. However, if a pre-AIA parent application is pending and an applicant inadvertently files a continuing application with claims having an effective filing date on or after March 16,2013, the applicant could file a continuation from the pre-AIA parent application without any claim to the benefit of the AIA application and without any claim to a claimed invention having an effective filing date on or after March 16, 2013. In this situation, the continuation would be examined as a pre-AIA application under pre-AIA 35 D.S.C. 102 and 103. Based on the facts presented, the parent application has not yet issued, so a continuation can be filed, making (D) the superior answer.

(A) is not correct because it is inferior in comparison to
(D). (A) is correct insofar as it goes, but the call of the question asks what advice do you give Jasper. Since there is a way that he can still have pre-AIA apply to at least the parent disclosure this advice, although correct, is incomplete. With a continuation filed relating to the parent application Jasper can still seek additional claims based off the original disclosure of the parent with pre-AIA law being applied. Thus, (D) is superior to (A).

(B) and (C) are incorrect because it is not too late to do anything. A continuation can be filed from the parent on or before April 15, 2014.

How well did you know this?
1
Not at all
2
3
4
5
Perfectly
20
Q

Samantha Rocks is the inventor of a new musical instrument called “the siren,” which produces enchanting, melodious tones like no other. With the assistance of a patent practitioner, Rocks filed a provisional patent application on January 5, 2014. Unfortunately, Rocks has fallen on hard times and can no longer afford the services of a patent practitioner. Undeterred, Rocks decides to file her own non provisional patent application, which was accomplished on January 15, 2015. The nonprovisional patent application was identical to the provisional filing.

Which of the following most accurately reflects the status of the January 15, 2015, nonprovisional patent application?

(A) The non provisional patent application will be considered a substitute application.
(B) The nonprovisional patent application will be considered timely filed because it was filed within 14 months of the filing of the provisional.
(C) The failure to preserve priority can be restored if the delay in filing the non provisional patent application was unintentional.
(D) If Rocks files a petition to extend the provisional patent application, she will be able to claim priority to the provisional in the January 15, 2015 filing.
(E) None of the above.

A

(C) is correct. Thanks to the Patent Law Treaty (PLT), applicants who unintentionally fail to file an application within the 12-month priority period can have that period extended up to an additional 2 months.

(A) is incorrect because the nonprovisional application is not a substitute application.

(B) is close, but not completely correct. Rocks will still need to seek restoration of the ability to claim priority and demonstrate that the delay in filing the nonprovisional patent application was unintentional. The PL T does not automatically make it possible to claim priority within 14 months.

(D) is incorrect because a provisional patent application only lasts 12 months and then goes abandoned. Provisional applications cannot be extended.

(E) is incorrect because (C) is correct

How well did you know this?
1
Not at all
2
3
4
5
Perfectly
21
Q

Sebastian Wilcox files a nonprovisional patent application on December 17, 2013. The application filed contains a specification sufficient to satisfy 35 U.S.c. 112(A), but does not include any claims. In due course, the Patent Office sends a Notice of Incomplete Application, indicating that a filing date has not been awarded. What should Wilcox do?

(A) Notify the Patent Office that pursuant to the Patent Law Treaty Implementation Act of 2012, a claim is no longer required at the time of filing in order to have a filing date awarded. Specifically request that a filing date of December 17, 2013, be awarded.
(B) Petition the Commissioner to award a filing date in light of the Patent Law Treaty Implementation Act of 2012.
(C) Convert the non provisional patent application into a provisional patent application because provisional patent applications do not require claims.
(D) Submit one or more claims in a Preliminary Amendment together with a request to retroactively grant December 17, 2013, as the official filing date.
(E) It is too late to do anything to fix this and salvage December 17, 2013, as a filing date. Therefore, Wilcox should file a substitute application as soon as possible.

A

(C) is correct. It is true that the Patent Law Treaty Implementation Act of 2012 (“PLT”) changes the law with respect to obtaining a filing date. Specifically, a claim is no longer required in order to have a filing date awarded. However, the provisions of the PL T that make it unnecessary to have a claim in order to obtain a filing date apply only to applications filed on or after December 18,2013. Because this application was filed on December 17,2013, a claim was required in order to obtain a filing date.

Since no claim was present, (A) and (B) are incorrect.

Likewise, (D) is incorrect because you cannot add a claim via Preliminary Amendment to obtain a retroactive filing date. Prior to the effective date of the PLT, a claim must be present in order to obtain a filing date. No retroactive filing dates are available.

(E) is incorrect because it is not too late to salvage December 17, 2013, as a filing date. You can still convert the nonprovisional patent application into a provisional patent application, which would salvage December 17, 2013, as a filing date … a filing date for a provisional patent application.

22
Q

Homer Sampson filed a nonprovisional patent application on October 27, 2011. The First Office Action on the Merits was mailed on June 16, 2013. Unfortunately, due to a rare medical condition, Sampson was, as his doctor explained, “largely out of his mind and wholly incapable of taking care of himself, let alone capable of making legal decisions.” After seeing a specialist on January 2, 2014, Sampson’s mAIAdy was properly diagnosed and treated. As Sampson was going through months of piled-up mail, he comes across the aforementioned Office Action. He comes to your offices on January 13, 2014, for assistance. You correctly notice that that the last day to respond to the Office Action within the statutory period was due on December 16, 2013, and that the application went abandoned on December 17, 2013. Which of the following do you tell him?

(A) He can petition the Office and revive the abandoned application as having been unavoidably abandoned.
(B) He can petition the Office and revive the abandoned application as having been unintentionally abandoned.
(C) Although the Patent Law Treaty Implementation Act of 2012 did away with petitions to revive for unavoidable abandonment, the changes did not become effective until December 18, 2013. Thus, since the application went abandoned on December 17,2013, it can still be revived as having been unavoidably abandoned.
(D) The Patent Law Treaty Implementation Act of 2012 did away with petitions to revive for unavoidable abandonment, but since the application was filed on October 27,2011, the changes brought about do not apply to this application. The application can be revived as having been unavoidably abandoned.
(E) This type of situation would have previously been considered an unavoidable abandonment and, since the Patent Law Treaty Implementation Act of 2012 did away with petitions to revive for unavoidable abandonment, there is nothing that can be done.

A

(B) is correct. Prior to the effective date of the Patent Law Implementation Treaty Act of 20 12 (“PL T”), this revival would have been based on the application being “unavoidably” abandoned. The PL T has done away with the distinction between “unavoidable” abandonment and “unintentional” abandonment by eliminating the “unavoidable” standard altogether. There is now a single standard (unintentional delay) and petition fee for reviving an abandoned application, accepting a delayed patent owner response in reexamination, and accepting a delayed maintenance fee payment.

(A) is incorrect because the PL T has eliminated the “unavoidable” standard. Although President Obama did not sign the PLT until December 18, 2012, and the changes do not take effect until December 18, 2013, (C) and (D) are incorrect because these changes to patent law brought about by the PLT apply to “any application for patent that is pending on or filed after the effective date …. “ Only the changes to what is required at the time of filing (i.e., a claim is no longer required) apply to applications filed on or after December 18,2013.

(E) is incorrect because, although the PLT eliminates the “unavoidable” standard, those abandonments that would have previously satisfied the “unavoidable” standard may still be addressed and revival sought by requesting revival as the result of abandonment being “unintentional.”

23
Q

Under the AIA, disclosures made one year or less before the effective filing date of a claimed invention may not be prior art. Which disclosures potentially qualify for this grace period and can be removed as prior art?

I. Printed publications.
II. Issued patents.
III. Public use.
IV. Offers for sale.

(A) I and II.
(B) III and IV.
(C) I, II and IV
(D) I, III and IV.
(E) I, II, III and IV.

A

(E) is correct. AIA 35 U.S.C. 102(A)(1) says: “A person shall be entitled to a patent unless- (1) the claimed invention was patented, described in a printed publication, or in public use, on sale, or otherwise available to the public before the effective filing date of the claimed invention …. “

The Office’s interpretation of AIA 35 U.S.C. 102(A)(1) is that the AIA grace period extends to all of the documents and activities enumerated in AIA 35 U.S.C. 102(A)(1) that would otherwise defeat patentability. This interpretation avoids the very odd potential result that the applicant who had made his invention accessible to the public for up to a year before filing an application could still obtain a patent, but the inventor who merely used his invention one day before he filed an application could not obtain a patent.

24
Q

Which of the following are required in order to obtain a filing date of a non provisional patent application at the United States Patent and Trademark Office?

(A) At least one claim.
(B) A specification describing the invention.
(C) A drawing if necessary to understand the invention.
(D) A, B and C.
(E) B and C.

A

(B) is correct. Thanks to the Patent Law Treaties Implementation Act of 2012, a claim is no longer required to file an application in the United States. Specifically, Sec. 201 of the Provisions to Implement the Patent Law Treaty explains “The filing date of an application shall be the date on which a specification, with or without claims, is received in the United States Patent and Trademark Office.” Sec. 201 also makes it clear, however, that ifat least one claim is not supplied in such time as the Director requires after filing, the application shall become abandoned. A specification is still required, but the USPTO has also said that drawings are no longer required to establish a filing date. They do point out, however, that drafting of claims and preparation of drawings at the time an application is prepared is the best practice and will help ensure that the application will contain adequate disclosure to satisfy 35 U.S.C. 112 and 35 U.S.C. 113.

25
Q

AIA 35 U.S.C. 102(A)(2) provides that a person is not entitled to a patent if the claimed invention was described in a U.S. patent, a published U.S. patent application, or an application for patent “deemed published” prior to the effective filing date of the claimed invention. Which of the following would be considered “deemed published” within the meaning of 102(A)(2)?

(A) A World Intellectual Property Organization (WI PO) publication of a Patent Cooperation Treaty (PCT) international application that designates the U.S.
(B) A World Intellectual Property Organization (WI PO) publication of a Patent Cooperation Treaty (PCT) international application that designates the United States, but only if subsequent to publication, the application enters the national stage in the U.S.
(C) A World Intellectual Property Organization (WI PO) publication of a Patent Cooperation Treaty (PCT) international application that designates the United States, but only if the application publishes in English.
(D) A World Intellectual Property Organization (WI PO) publication of a Patent Cooperation Treaty (PCT) international application that designates the U.S, but only if the application matures into an issued U.S. patent.
(E) None of the above.

A

(A) is correct. 35 U.S.C. 122(B). A World Intellectual Property Organization (WIPO) publication of a Patent Cooperation Treaty (PCT) international application that designates the United States is an application for patent deemed published under 35 U.S.C. 122(B) for purposes of AIA 35 U.S.C. 102(A)(2). Thus, under the AIA, WIPO publications of PCT applications that designate the United States are treated as U.S. patent application publications for prior art purposes, regardless of the international filing date, whether they are published in English, or whether the PCT international application enters the national stage in the United States. Accordingly, a U.S. patent, a U.S. patent application publication, or a WIPO publication of a PCT application (WIPO published application) that designates the United States, that names another inventor and was effectively filed before the effective filing date of the claimed invention, is prior art under AIA 35 U.S.C. 102(A)(2).

26
Q

Jocelyn Carter is the inventor of a unique and highly concealable spy camera, which incorporates facial recognition modules. Carter files a non provisional patent application on April 2, 2012. The nonprovisional patent application does not contain any additional disclosure, but is filed with 21 claims, 4 of which are independent claims. On March 14, 2013, a Continuation-in-Part (CIP) is filed, further disclosing a previously undisclosed computer-implemented filtering system for sorting the recognized faces. On March 17, 2013, a continuation of the April 2, 2012, nonprovisional patent application is filed.

Which of the following accurately states which law these applications will be examined under?

I. The April 2, 2012, nonprovisional patent application will be reviewed with the examiner applying pre-AIA.
II. The March 14, 2013, CIP will be reviewed with the examiner applying pre-AIA.
III. The March 17, 2013, Continuation will be reviewed with the examiner applying AIA.

(A) l
(B) II
(C) I and II
(D) I and III
(E) I, II and III

A

(C) is correct. (I) is correct because the nonprovisional patent application filed on April 2, 2012, will be examined under pre-AIA because it was filed prior to March 16, 2012. (II) is correct because the CIP will be treated under pre-AIA. Although it contains a new embodiment not found in either the provisional or nonprovisional applications, it was filed right before the trigger date for AIA application. Applications filed on or after March 16,2013, are the first that are potentially open for treatment under the AIA. (III) is incorrect because the continuation filed on March 17,2013, will be treated under preAIA because as a continuation the disclosure is identical to the parent, which in this case was filed on April 13,2012. Thus, even though it was filed on or after March 16,2013, it is entitled to pre-AIA based on its priority date.

27
Q

Stanley Thaddeus Wojciehowicz is the inventor of an improved, combination coffeemaker/motion detector, which is useful for making better than mediocre coffee and doubles as a home security system. He first files a patent application in Poland on February 28,2012. Subsequently, he files a patent application in the U.S. on February 27, 2013, which claims the benefit of his Polish filing date. The Polish application publishes on August 28, 2013, and the U.S. patent application publishes on August 29, 2013. The Polish patent is granted on March 14, 2014, and the U.S. patent is granted on October 22, 2014.

On November 1, 2014, Carl Levitt files an application claiming a combination coffee maker/motion detector with integrated radio.

Which of the follow dates is the earliest date the Wojociehowicz disclosure will be prior art to Levitt?

(A) February 28, 2012.
(B) February 27, 2013.
(C) August 28, 2013.
(D) March 14, 2014.
(E) October 22, 2014.

A

(A) is correct. AIA 35 U.S.C. 102(D) provides that if the U.S. patent, U.S. patent application publication, or WIPO-published application claims priority to one or more prior-filed foreign or international applications under 35 U.S.C. 119 or 365, the patent or published application was effectively filed on the filing date of the earliest such application that describes the subject matter. Therefore, if the subject matter relied upon is described in the application to which there is a priority or benefit claim, a U.S. patent, a U.S. patent application publication, or WIPO-published application is effective as prior art as of the filing date of the earliest such application, regardless of where filed. Thus, the Polish filing date of February 28, 2012, is the earliest date the Wojociehowicz disclosure will be prior art to Levitt.

28
Q

Justin Kindle is the inventor of a new and improved weed-cutting golf club. The club has a sharp edge which, when swung with force, will cut weeds and tall grass, thereby allowing the user to keep and maintain a healthy lawn while at the same time allowing the user to work on their golf game. Kindle, who is very proud of his invention, writes up a description of the invention together with several drawings, and shows them to various individuals at his country club, all of whom like the invention. The first such disclosure occurred on March 15, 2013. One fellow by the name of Eric Manning likes it so much that he decides to publish the description and drawings he obtained from Kindle on his widely read website, which occurs on March 17,2013. Kindle subsequently files a patent application on July 29, 2013. The application filed contains claims to the version of the golf club disclosed by Kindle previously, as well as claims directed to a previously undisclosed embodiment that incorporates a GPS tracking system that analyzes the trajectory of the swing and offers constructive feedback on swing mechanics.

Will it be possible for Kindle to overcome a rejection by a patent examiner based on the Manning publication?

(A) Yes. Kindle can submit a Rule 1.130 affidavit.
(B) Yes. Kindle can submit a Rule 1.131 affidavit.
(C) Yes. Kindle can submit a Rule 1.132 affidavit.
(D) Yes, but Kindle will be required to file a petition to institute a derivation proceeding under AIA 35 u.s. c. 135.
(E) No, Kindle will not be able to remove the reference under the AIA first-to-file rules.

A

(A) is correct. Unless the other party (the suspected deriver) has submitted his or her own application, the issue for the applicant is disqualifying the prior art under AIA 35 U.S.c. 102(B) rather than showing derivation under AIA 35 U.S.C. 135. If the prior art disclosure was made one year or less before the effective filing date of the claimed invention, the applicant may submit an affidavit or declaration under 37 CFR 1.130 to show that the disclosure was by a party who obtained the subject matter disclosed directly or indirectly from the inventor or a joint inventor, thereby disqualifying the prior art under 35 U.S.C. 102(B)(1)(A). Therefore, (A) is correct and (D) is incorrect.

(B) is incorrect because a 131 affidavit applies pre-AIA where the applicant is attempting to establish an earlier date of conception.

(C) is incorrect because a 131 affidavit is one that you file to submit evidence to the Patent Office. It is not the correct procedural vehicle to remove a reference either under AIA or pre-AIA.

(E) is incorrect because you can use a 130 affidavit in this scenario.

29
Q

Michael Arthur is the inventor of a cordless jump rope. Essentially, the invention is a jump-rope simulator incorporating rotatable, counterbAIAncing weights within a handle and marketed to those individuals who really like to jump rope but who lack the physical prowess necessary to actually jump the rope. Arthur, who is very proud of his invention, writes up a description of the invention together with several drawings and shows them to various individuals at his health club, all of whom like the invention. The first such disclosure occurred on June 15, 2013. One fellow by the name of Bradley Brady likes it so much that he decides to file a patent application on the cordless jump rope, which occurs on November 15, 2013. Arthur subsequently files a patent application on December 15, 2013. The application filed by Brady fully discloses the Arthur invention. The application filed by Arthur includes 20 claims, 3 of which are in independent format. The claims filed by Arthur are quite different compared to the claims submitted by Brady.

Will it be possible for Arthur to overcome a rejection by a patent examiner based on the Brady application?

(A) Yes. Arthur can submit a Rule 1.130 affidavit.
(B) Yes. Arthur can submit a Rule 1.131 affidavit.
(C) Yes. Arthur can submit a Rule 1.132 affidavit.
(D) Yes, but Arthur will be required to file a petition to institute a derivation proceeding under AIA 35 U.S. C. 135.
(E) No, Arthur will not be able to remove the reference under the AIA first-to-file rules.

A

(A) is correct. Unless the other party (the suspected deriver) has submitted his or her own application, the issue for the applicant is disqualifying the prior art under AIA 35 U.S.C. 102(B) rather than showing derivation under AIA 35 U.S.C. 135. Here, Brady has filed a patent application based on Arthur’s invention, but AIA 35 U.S.C. 135 applies to a claim to an invention that is the “same or substantially the same” as a claim of an earlier application. Thus, because Arthur’s claims are quite different compared to the claims submitted by Brady, Arthur will not need to file a petition to institute a derivation proceeding and instead can utilize a 130 affidavit to avail himself of the exception pursuant to AIA 35 U.S.C. 102(B). Thus, (A) is correct and (D) is incorrect.

(B) is incorrect because a 131 affidavit applies pre-AIA where the applicant is attempting to establish an earlier date of conception.

(C) is incorrect because a 131 affidavit is one that you file to submit evidence to the Patent Office. It is not the correct procedural vehicle to remove a reference either under AIA or pre-AIA.

(E) is incorrect because you can use a 130 affidavit in this scenario.

30
Q

Spencer Kelly is the inventor of an attachment to a shotgun barrel, which modifies the pattern of the shot released from the shotgun shell when it is fired. Kelly, who is very proud of his invention, writes up a description of the invention together with several drawings and shows them to various individuals at his gun club, all of whom like the invention. The first such disclosure occurred on August 22, 2013. One fellow by the name of Alex Lexington likes it so much that he decides to file a patent application on the shotgun barrel attachment, which occurs on October 22, 2013. Kelly subsequently files a patent application on January 1, 2014. The application filed by Lexington fully discloses the Kelly invention. The application filed by Kelly includes 25 claims, 5 of which are in independent format. Claims 1- 5 filed by Kelly are identical to claims submitted by Lexington, Claims 6 - 10 filed by Kelly are substantially similar to the claims filed by Lexington, and Claims 11- 25 are quite different when compared with any claims filed by Lexington.

Will it be possible for Kelly to overcome a rejection by a patent examiner based on the Lexington application?

(A) Yes. Kelly can submit a Rule 1.130 affidavit to overcome any rejection of claims 1- 25.
(B) Yes. Kelly can submit a Rule 1.130 affidavit to overcome any rejection of claims 11- 25.
(C) Yes. Kelly can submit a Rule 1.131 affidavit.
(D) Yes. Kelly can submit a Rule 1.132 affidavit.
(E) Yes, but Kelly will be required to file a petition to institute a derivation proceeding under AIA 35 U.S. C. 135.

A

(B) is the best answer. When a suspected deriver has submitted his or her own application, the critical question turns to whether the claims submitted are the “same or substantially the same” as a claim of an earlier application. If the claims submitted by Kelly are the same or substantially the same when compared to the claims filed by Lexington, Kelly will be required to pursue a derivation proceeding. If the claims are not the same or substantially similar to the claims filed by Lexington, then Kelly can take advantage of the AIA 35 U.S.C. 102(B) exception by utilizing a 130 affidavit. This question presents a mixture of the two. Claims 1 - 10 filed by Kelly are either identically or substantially similar to those filed by Lexington. Thus, Kelly cannot resort to a 130 affidavit with respect to those claims and must pursue a derivation proceeding. Claims 11 - 25 are, however, quite different than those claims submitted by Lexington; thus, a 130 affidavit can be used to remove the Lexington reference as prior art for those claims. Thus, (B) is correct and (A) is incorrect.

(C) is incorrect because a 131 affidavit applies to pre-AIA where the applicant is attempting to establish an earlier date of conception.

(D) is incorrect because a 132 affidavit is one that you file to submit evidence to the Patent Office. It is not the correct procedural vehicle to remove a reference either under AIA or pre-AIA.

(E) is incorrect because it is overbroad. A derivation proceeding is not required if Kelly wishes to pursue claims 11 - 25 because those claims are not the same or substantially similar to claims previously filed by Lexington. So (E) isn’t completely wrong, but (B) is a better answer.

31
Q

Casper Wright is the inventor of a timed drinking vessel, which incorporates a digital timer on the front of a beverage container, such as a beer mug, for example. The invention further includes a strong magnet that engages and prevents the container from being lifted from a metal surface until a predetermined appropriate amount of time has passed. The predetermined amount of time is determined via a computerimplemented process that takes into account the alcohol content of the beverage and the weight of the individual consuming the beverage. The purpose of the invention is to make sure that the consumer of the beverage never drinks too much too fast, or at least if they do, they will be well aware of that fact. The invention is conceived on January 15, 2013, and reduced to practice on May 23, 2013. A provisional patent application that fully describes the invention is filed on December 24, 2013. A nonprovisional patent application, which is identical to the provisional patent application, is filed on March 14, 2014. The examiner rejects the Wright claims based on a patent issued to Stein, which issued on September 14, 1878, in combination with a patent covering a digital watch issued to Quartz on October 18, 1982, further in view of a magnetized, computer locking device for a handgun, which patented on December 15, 2013, based on an accelerated application filed April 3, 2013. Which, if any, of the following statements is correct relative to this obviousness rejection?

(A) Obviousness is determined based upon whether the claimed invention would have been obvious to one of skill in the art at the time of conception.
(B) Obviousness is determined based upon whether the claimed invention would have been obvious to one of skill in the art as of the effective filing date of the claimed invention.
(C) Obviousness is determined based upon whether the claimed invention would have been obvious to one of skill in the art at the time of the filing of the nonprovisional patent application.
(D) Obviousness is determined based upon whether the claimed invention would have been obvious to one of skill in the art as of the date the invention was reduced to practice.
(E) None of the above.

A

(B) is correct. The most significant difference between the AIA 35 U.S.C. 103 and pre-AIA 35 U.S.c. 103(A) is that AIA 35 U.S.C. 103 determines obviousness as of the effective filing date of the claimed invention, rather than as of the time that the claimed invention was made. Under pre-AIA examination practice, the Office used the effective filing date as a proxy for the invention date, unless there is evidence of record to establish an earlier date of invention. Ultimately, however, the question under pre-AIA was whether the claimed invention was obvious as of the date of conception of the invention. Thus, (B) is correct and (A) is incorrect.

(C) is incorrect because the critical time to make the obviousness determination is as of the effective filing date, which in this case would relate back to the filing of the provisional patent application on December 24,2013.

(D) is incorrect under both AIA and pre-AIA because reduction to practice is not the proper time to determine whether a claimed invention is obvious under either AIA or pre-AIA.

(E) is incorrect because (B) is correct.

32
Q

The University of Akron and Goodyear are parties to a contract, dated January 3, 2012. This agreement is for the performance of experimental, developmental and/or research work relating to certain high-strength polymers. As a part of the contract, the parties share laboratory space, share research, and scientists and engineers collaborate in a cooperative manner. The agreement explains that any patents will be co-owned only as to claims in the patent attributable to both Akron and Goodyear inventors. Otherwise, patents will be individually owned, with the other party obtaining a nonexclusive license.

On January 10, 2014, Akron applies for a patent on behalf of Joseph Anthony, a Chemical Engineer employed by Akron, who is the sole inventor. The invention was made using laboratory space provided by Goodyear and relying on basic research contributed by Goodyear. The patent examiner rejects all of the claims based upon an earlier-filed patent application filed jointly by Akron and Goodyear, which discloses certain characteristics of the high-strength polymer claims in the Anthony application. Can Akron overcome this rejection?

(A) Yes, because the claimed invention was made after execution of the agreement with Goodyear.
(B) Yes, because the claimed invention was made as a result of activities undertaken within the scope of the joint research agreement.
(C) Yes, provided that the Anthony application discloses, or is amended to disclose, the names of the parties to the joint research agreement.
(D) Yes. The claimed invention was made after execution of the agreement with Goodyear. Thus, as long as the application discloses or is amended to disclose the names of the parties to the joint research agreement, Akron can overcome the rejection.
(E) Yes. The claimed invention was made after execution of the agreement with Goodyear and was the result of activities undertaken within the scope of the agreement. Therefore, as long as the application discloses or is amended to disclose the names of the parties to the joint research agreement, Akron can overcome the rejection.

A

(E) is correct. AIA 35 U.S.C. 102(C) discusses the scenario of common ownership under joint research agreements. First, the AIA defines the term “joint research agreement” as a written contract, grant, or cooperative agreement entered into by two or more persons or entities for the performance of experimental, developmental, or research work in the field of the claimed invention. When such an agreement is in place, prior art can be removed if three conditions are satisfied. First, the subject matter disclosed must have been developed and the claimed invention must have been made by, or on behalf of, one or more parties to a joint research agreement that was in effect on or before the effective filing date of the claimed invention. Second, the claimed invention must have been made as a result of activities undertaken within the scope of the joint research agreement. Third, the application for patent for the claimed invention must disclose, or be amended to disclose, the names of the parties to the joint research agreement.
(A), (B) and (C) are incorrect because each focuses on only one prong of the three conditions that all must be present.

(D) is incorrect because it weaves together the first and third condition, but does not mention that the claimed invention must also have been made as the result of activity contemplated by the joint research agreement.

33
Q

Independently of each other, Bert and Ernie invented the same rubberized, floating toy in the United States. A U.S. Patent was granted to Bert on February 18, 2012 on an application filed on April 12, 2010, claiming the toy. On April 10, 2012, Ernie filed a patent application in the USPTO claiming the same toy. There is no common assignee. Under which of the following provisions of 35 U.S.C. 102 is the U.S. patent to Bert prior art with regard to the toy claimed by Ernie?

(A) 35 U.S.C. 102(A)
(B) 35 U.S.C. 102(B)
(C) 35 U.S.C. 102(E)
(D) A and C
(E) A, B, and C

A

(D) is correct. Pre-AIA 102. Based on the dates, we are dealing with pre-AIA 102. The facts make out a prima facie case under 102(A) and 102(E). The l02
(E) rejection is based on Bert’s filing date, which is two years prior to Ernie’s filing date. The 102(A) rejection is supported because the same invention was already patented in the US before invention by Ernie. This is true because Ernie’s date of invention is presumed to be his filing date and there are no facts in the question to support an earlier date of conception, let alone one that would overcome an
(A) or an
(E) rejection.

34
Q

Independently of each other, Oscar and Felix invented identical ergonomic golf clubs in the United States. Oscar filed his patent application in the U.S. on April 12, 2012, and a patent issued on March 12, 2013. On April 10, 2013, Felix filed his patent application in the USPTO claiming golf clubs identical to those claimed in Oscar’s patent. There is no common assignee. Under which of the following provisions of 35 U.S.C. 102 is the U.S. patent to Oscar prior art with regard to the tennis racket claimed by Felix?

(A) 35 U.S.C. 102(A)
(B) 35 U.S.C. 102(B)
(C) 35 U.S.C. 102(E)
(D) A and C
(E) A, B, and C

A

(A) is correct. AIA 102(A)(1). Based on the filing date of Felix’s application, the law that will apply to Felix is AIA 102. AIA 102(A)(1) prevents a patent when the claimed invention was patented (in this case by Oscar) before the effective filing date of the claimed invention. Thus, Felix will have his claim rejected under 102(A)(I).

(B) is incorrect because AIA 102(B) is not a basis for a rejection, but rather details exceptions.

(C) is incorrect because there no longer is a section 102(E) in AIA 102.

(D) and (E) are similarly incorrect.

35
Q

Which of the following references could be used by an examiner to appropriately reject claims in a patent application filed by Jonathan Williamson under 35 U.S.C. 102(A)(2)? Assume Williamson conceived of his invention on March I, 2013, and filed a patent application fully disclosing the invention on October I, 2013.

(A) An article in Science Daily describing the invention in detail, which was authored by Williamson and published on August I, 2013.
(B) An article in Scientific American describing the invention in detail, which was authored by Williamson and his colleague Carter Thomas and published on September IS, 2013.
(C) An Irish patent application publication that lists Williamson as the inventor and which published on November I, 2013, and with the Irish filing date relied upon to support a priority claim for a PCT application that published.
(D) A French patent application that lists Williamson and Carter Thomas as inventors, which was filed on June 1, 2012 and which was later relied upon to support a proper priority claim for a PCT application that published.
(E) None of the above.

A

(D) is correct. AIA 35 U.S.C. 102(A)(2) states: “A person shall be entitled to a patent unless- the claimed invention was described in a patent issued under section 151, or in an application for patent published or deemed published under section 122(B), in which the patent or application, as the case may be, names another inventor and was effectively filed before the effective filing date of the claimed invention.”

The key to getting this question right is to know what 102(A)(2) captures as prior art. By the explicit terms of this section of the statute, 102(A)(2) captures prior-filed U.S. patent applications, as well as prior-filed patent applications that are deemed published, provided however that they must name “another inventor.” We can rule out (A) and (B) because these relate to publications. (C) can be ruled out because the only inventor is Williamson, who is also the inventor on the U.S. patent application in question. (D) lists Williamson and Thomas as co-inventors, which satisfies the “another inventor” requirement of 102(A)(2). The French filing date will be considered a prior art date because it was relied upon in a published PCT application. Notice that the fact that the French patent application will not publish until after Williamson’s U.S. filing date does not matter since it is the French application date that creates the problem.

The key here is to notice that, if there is any difference in inventive entity between the application under examination on the one hand and either a prior art U.S. patent, U.S. patent application publication or WIPO-published PCT application, the U.S. patent, U.S. patent application publication, or WIPO-published PCT application satisfies the “names another inventor” requirement of AIA 35 U.S.C. 102(A)(2). Thus, in the case of joint inventors, only one inventor needs to be different for the inventive entities to be different.

36
Q

35 U.S.C. 102(B)(1)(B) provides that a grace period disclosure shall not be prior art to a claimed invention under AIA 35 U.S.C. 102(A)(1) if the subject matter disclosed had, before such disclosure, been publicly disclosed by the inventor or a joint inventor. In order to remove a reference and take advantage of this exception, which of the following does an inventor have to do in a Rule 1.130 Affidavit?

I. Identify the subject matter publicly disclosed and establish the date and content of their earlier public disclosure.
II. If the earlier public disclosure was a printed publication, the affidavit or declaration must be accompanied by a copy of the printed publication.
III. Demonstrate that the prior public disclosure by an inventor or a joint inventor was disclosed in the same manner and using the same or similar key terms and phrases.
IV. Demonstrate that the disclosure by the inventor, a joint inventor, or another who obtained the subject matter disclosed directly or indirectly from an inventor or a joint inventor was an enabling disclosure of the subject matter within the meaning of 35 U.S.C. 112(A).

(A) I and IV.
(B) II and III.
(C) I and II.
(D) I and III.
(E) I, II and IV.

A

(C) is correct. I and II are the only correct statements. More specifically, an applicant may show that the subject matter disclosed had been publicly disclosed by the inventor or a joint inventor before the disclosure or effective filing date of the subject matter on which the rejection was based by way of an affidavit or declaration under 37 CFR 1.130(B) (an affidavit or declaration of prior public disclosure). Specifically, the affidavit or declaration must identify the subject matter publicly disclosed and establish the date and content of their earlier public disclosure. If the earlier public disclosure was a printed publication, the affidavit or declaration must be accompanied by a copy of the printed publication in accordance with 37 CFR 1.130(B)(1). If the earlier disclosure was not a printed publication, the affidavit or declaration must describe the earlier disclosure with sufficient detail and particularity to determine that the earlier disclosure is a public disclosure of the subject matter, as required by 37 CFR 1.130(B)(2).

III is incorrect. When using an affidavit or declaration under 37 CFR 1.130(B) to disqualify an intervening disclosure as prior art based on a prior public disclosure by an inventor or a joint inventor, it is not necessary for the subject matter to have been disclosed in the same manner or using the same words. This difference in the manner of disclosure or differences in the words used to describe the subject matter will not preclude the inventor from submitting an affidavit or declaration under 37 CFR 1.130(B) to disqualify the intervening disclosure (e.g., a journal article) as prior art. IV is incorrect. An affidavit or declaration under 37 CFR 1.130 need not demonstrate that the disclosure by the inventor, a joint inventor, or another who obtained the subject matter disclosed directly or indirectly from an inventor or a joint inventor was an “enabling” disclosure of the subject matter within the meaning of35 U.S.C. 112(A). Rather, an affidavit or declaration under 37 CFR 1.130 must show that: (1) The disclosure in question was made by the inventor or a joint inventor, or the subject matter disclosed was obtained directly or indirectly from the inventor or a joint inventor (37 CFR 1.l30(A)); or (2) the subject matter disclosed had, before such disclosure was made or before such subject matter was effectively filed, been publicly disclosed by the inventor or a joint inventor or another who obtained the subject matter disclosed directly or indirectly from the inventor or a joint inventor (37 CFR 1.130(B)).

37
Q

In which of the following circumstances would it not be possible for an applicant who filed on December 1, 2013, to rely on a Rule 1.130 affidavit to take advantage of the exceptions of 102(B)(1)?

(A) The inventor publicly disclosed the subject matter in question via a slide presentation at a scientific meeting on January 15, 2013, while the intervening disclosure of the subject matter was made in a journal article on September 1, 2013.
(B) The party making the intervening disclosure learned of the subject matter in question after attending a lecture given by the inventor on October 1, 2012, and subsequently published an article detailing the inventor’s invention on December 15,2012.
(C) The party making the intervening disclosure learned of the subject matter in question after attending a lecture given by the inventor on April 1, 2013, and subsequently published an article detailing the inventor’s invention on November 15,2013.
(D)The party making the intervening disclosure learned of the subject matter in question after seeing a draft of an academic article provided to him for purpose of peer review. The draft was circulated on May 1, 2013, and an article was subsequently published detailing the inventor’s invention as described in the draft article on November 30, 2013.
(E) None of the above.

A

(B) is correct. AIA 35 U.S.C. 102(B)(1)(B) provides that a disclosure which would otherwise qualify as prior art under AIA 35 U.S.C. 102(A)(1) [i.e., a patent, printed publication, public use, sale, or other means of public availability) may be disqualified as prior art if: (1) The disclosure was made one year or less before the effective filing date of the claimed invention; and (2) the subject matter disclosed had been previously publicly disclosed by the inventor, a joint inventor, or another who obtained the subject matter directly or indirectly from the inventor or joint inventor. The previous public disclosure of the subject matter by the inventor, a joint inventor, or another who obtained the subject matter directly or indirectly from the inventor or joint inventor must itself be a disclosure within the one-year grace period.

In (A), (C) and (D), the initial disclosure occurred within one year of filing the patent application.

In (B), however, the initial disclosure happened some 14 months prior to the filing of the patent application on December 1, 2013. Thus, the applicant could not rely on a Rule 1.130 affidavit. The applicant would not be able to remove their own disclosure of October 1,2013.

38
Q

In addition to the above facts of Questions 9, John filed a patent application in Canada in April of 2010, which issued as a patent in December 2011. John now believe his previously filed non provisional patent application is defective beyond repair and wants to file a new non provisional patent application in the US that does not claim priority to the previously filed US nonprovisional application.

Which of the following sections of pre-AIA 102 bar patentability as of today, March 1, 2013?

(A) 102(C)
(B) 102(D)
(E) 102(E)
(D) 102(g)
(E) None of the above.

A

(B) is correct because all of the requirements of pre-AIA 102(D) are satisfied. John filed a patent application in Canada more than 12 months before he filed a patent application in the U.S. Further, the Canadian patent issued prior to his U.S. filing. Since he is the inventor on both the Canadian and U.S. patent applications and because the invention is the same, all of the pre-AIA 102(d) requirements are met.

(A) is incorrect because there are no facts that suggest John abandoned the invention. Quite to the contrary, he has been actively trying to obtain patents.

39
Q

In addition to the above facts of Question 9, a U.S. patent issued to Jones in August 2012, which was filed in 2009 and which fully describes, but does not claim, the balloon. Based on these additional facts, which of the following sections of pre-AIA 102 bar patentability?

(A) 102(C)
(B) 102(D)
(C) 102(E)
(D) 102(g)
(E) None of the above.

A

(C) is correct because the Jones patent was filed in 2009. The Jones patent becomes prior art as of the filing date, which is a date that John cannot get behind.

(A) is incorrect because there are no facts that suggest John abandoned the invention. Quite to the contrary, he has been actively trying to obtain patents.

(B) is incorrect because there is no mention of a foreign patent or patent application in this question, which are pre-requisites for there to even possibly be a pre-AIA 102(D) rejection.

(D) is incorrect because pre-AIA 102(g)(l) is the section responsible for determining which applicant of identical claims will receive the patent under first-to-invent principles. There is no evidence of an interference proceeding in this question. Pre-AIA 102(g)(2) relates to a situation where the invention was made in the U.S. prior to the date of invention by the applicant for patent. Pre-AIA 102(g)(2) specifically says: “A person is entitled to a patent unless - before such person’s invention thereof, the invention was made in this country by another inventor who had not abandoned, suppressed, or concealed …. “ There are no facts in the question to support a valid pre-AIA 102(g)(2) rejection because we do not know where or whether the Jones invention was ever actually made.

40
Q

Which of the following sections of pre-AIA 102 can be the basis for a provisional rejection?

(A) 102(A)
(B) 102(B)
(E) 102(E)
(D) 102(g)
(E) 102(A) and 102(B).

A

(C) is correct because a provisional rejection can only be made pursuant to pre-AIA 102(E). It is provisional in nature because a pre-AIA 102(E) date does not exist unless a patent application has published or a patent has issued. If there is a common owner or common inventor, then the patent examiner may issue a provisional pre-AIA 102(E), which is a notice that once a pre-AIA 102(e) date exists, it will be used to reject the claim or claims at issue.

(A), (B), (D) and (E) are incorrect because under these provisions, there is no ability or justification to issue a provisional or preliminary rejection.

41
Q

The answer to each of Questions 4 and 5 are based on the facts in the following paragraph. You are to consider each of the questions independently of the others.

Mary Rose can prove she conceived and reduced her invention to practice respectively on February 27, 2011 and on August 20, 2011 in the United States. On March 14, 2012 she filed a provisional application completely describing her invention and on March 13, 2013 she filed a utility application in the United States claiming the benefit of her provisional filing.

For each of the following facts, determine which section, if any, of pre-AIA 102 applies.

  1. Unbeknownst to Mary, General Mega Corporation filed a patent application in Belgium on September 30, 2011, which issued on March 15, 2012.
    (A) 102(A)
    (B) 102(B)
    (C) 102(D)
    (D) 102(E)
    (E) None of the above.
  2. Mary’s invention was used in public, by Smith, who learned about the invention from Mary, in Maine without Mary’s permission on August 23, 2012.
    (A) 102(A)
    (B) 102(B)
    (C) 102(C)
    (D) 102(E)
    (E) None of the above.
A
  1. (E) is correct because, based on these facts, there is no valid rejection that can be issued.

(A) is incorrect because pursuant to pre-AIA 102(A), the Belgian patent/application would not be prior art until it is published or issued. Mary can get behind the March 15,2012 issue date of the Belgium patent with an earlier date of conception and a provisional patent application filed on March 14,2012.

(B) is incorrect because pursuant to pre-AIA 102(A), the Belgian patent/application would not be prior art until it is published or issued. Mary can get behind the March 15,2012 issue date of the Belgium patent with an earlier date of conception and a provisional patent application filed on March 14,2012.

(C) is incorrect because a pre-AIA 102(d) rejection is appropriate where the same inventor is seeking a patent in the U.S. on the same invention after no longer being able to claim foreign priority and after the patent application has already issued. Pre-AIA 102(d) is a statutory bar. The patent or application of another cannot be used to reject an applicant under pre-AIA 102(D).

(D) is incorrect because a foreign patent application does not receive a 102(e) date. Thus, the Belgian filing date is not a prior art date. Of course, on these facts, even if it were, Mary would still be able to get behind with an earlier date of invention.

  1. (E) is correct because, based on these facts, there is no valid rejection that can be issued.

(A) is incorrect because pre-AIA 102(A) deals with patents, publications and knowledge or use within the U.S. prior to the date of invention by the applicant. Here, Smith used the invention publicly but not until after Mary had already filed a patent application and long after Mary conceived.

(B) is incorrect because pre-AIA 102(B) deals with patents, publications, public use and offers for sale more than 12 months before the filing of a U.S. patent application. The public use by Smith occurred only after Mary filed her patent application; thus, there is no pre-AIA 102(B) issue.

(C) is incorrect because pre-AIA 102(C) deals with abandonment of the invention. There is nothing in this question to suggest the invention has been abandoned.

(D) is incorrect because pre-AIA 102(E) deals with whether there is a filing date that can be considered prior art. Here, we are not dealing with another application, but public use by Smith after Mary already filed her patent application. Thus, there is no pre-AIA 102(E) issue.

42
Q

Pre-AIA 102(D) requires that:
(A) A foreign application be filed more than twelve months before a U.S. application (six months in the case of a design application).
(B) A foreign application be filed more than twelve months before a U.S. (six months in the case of a design application) and the foreign patent has issued.
(C) A foreign application can be filed more than twelve months before a U.S. (six months in the case of a design application) and the foreign patent has been published.
(D) The foreign application has exactly the same claims as the U.S. application.
(E) A foreign filing license be obtained.

A

(B) is correct because pre-AIA 102(D) requires four things: (1) the same applicant; (2) same invention; (3) in the case of a utility application. that the foreign filing occurred more than 12 months earlier than the u.s. filing; and (4) that the foreign patent has already issued. Remember, in the case of a design patent, criteria (3) is reduced to 6 months.

(A) is incorrect because it is incomplete. The mere fact that a foreign patent application was filed so long ago that foreign priority cannot be claimed does not in and of itself render a claimed invention unpatentable. The foreign patent must also have been granted prior to the U.S. filing.

(C) is incorrect because the trigger is not the publication of the foreign patent, but rather the date it is granted.

(D) is also incorrect because the fact that a foreign application has the same claims as the U.S. application is irrelevant under pre-AIA 102(D).

(E) is incorrect because it has nothing to do with a foreign filing license. It is a substantive provision for evaluating whether a claim in a patent application is considered novel.

43
Q

Pre-AIA 102(g)(2) requires:
(A) That the prior invention not be abandoned, suppressed or concealed.
(B) That the prior invention be made in this country.
(C) That a prior invention has been made.
(D) That the invention be by another.
(E) All of the above are true.

A

(E) is correct is correct because (A), (B), (C) and (D) are all true statements. Pre-AIA 102(g)(I) is the section that determines which applicant of identical claims will receive the patent under first-to-invent principles. Pre-AIA 102(g)(2) relates to a situation where the invention was made in the U.S. prior to the date of invention by the applicant for patent. Pre-AIA 102(g)(2) specifically says: “A person is entitled to a patent unless - before such person’s invention thereof, the invention was made in this country by another inventor who had not abandoned, suppressed, or concealed …. “ Thus, pre-AIA 102(g)(2) is a novelty section, which evaluates whether another invented prior to the applicant for patent.

44
Q

Which of the following qualify as prior art under pre-AIA 1027
(A) An offer for sale of the rights to the invention.
(B) An offer for sale of the invention in Japan.
(E) An abandonment of the invention.
(D) An unpublished Japanese application.
(E) A confidential disclosure to a potential licensee.

A

(C) is correct because an abandonment of the invention is prior art. PreAIA 102(C) says: “A person is entitled to a patent unless - he has abandoned the invention.”

(A) is incorrect because the offer for sale of the rights to an invention is not a prior art event under pre-AIA 102.

(B) is incorrect because offers for sale of the invention in Japan are not prior art events under pre-AIA 102. To be prior art under pre-AIA 102(B), an offer for sale must occur in the U.S.

(D) is incorrect because an unpublished Japanese patent application would not qualify as prior art under any pre-AIA 102 section.

(E) is incorrect because a confidential disclosure to a potential licensee would not qualify as prior art under any pre-AIA 102 section.

45
Q

John Key invented a new and unique air balloon in Canada in 2010. He reduced the invention to practice in Canada and filed a nonprovisional U.S. application on June 14, 2011, which published on December 20, 2013. In May of 2010, he sold a few of the balloons in Canada. Which of the following sections of pre-AIA 102 bar patentability?

(A) 102(A)
(B) 102(B)
(E) 102(E)
(D) 102(g)
(E) None of the above.

A

(E) is correct because, on these facts, there is nothing that will prevent John from obtaining a patent.

(A), (C) and (D) are all incorrect because there is nothing in these facts to suggest a valid pre-AIA 102 rejection based on a novelty section. PreAIA 102(A), (E) and (g) would all require an action by another. There are no other people identified in the question. Thus, since the inventor cannot create their own problem under
(A), (E) or (g), these cannot be the right answer.

(B) is incorrect because John did not start selling the balloons until May 2010, but even then the sale was in Canada, which would not trigger preAIA 102(B). Had the sale occurred in the U.S., John would have been in trouble because he did not file a patent application until 13 months after the sale.

46
Q

In addition to the facts of Question 9, John also published a catalog in Canada fully describing the balloon in May, 2010. Based on these additional facts, which of the following sections of pre-AIA 102 bar patentability?

(A) 102(C)
(B) 102(B)
(E) 102(E)
(D) 102(g)
(E) None of the above.

A

(B) is correct because the invention was fully described in a published catalog in May 2010, which was more than 12 months before John filed a U.S. patent application.

(A) is incorrect because there are no facts to suggest that John abandoned the invention.
(C) and (D) are incorrect because there are no other individuals or entities mentioned in the question. The inventor cannot create their own problem under pre-AIA 102(E) or (g) because they are novelty sections that ask whether someone else invented prior to the inventor.

(E) is incorrect because (B) is correct.

47
Q

Which of the following sections of pre-AIA 102 do not require that the act be “by another”?

(A) 102(A)
(B) 102(B)
(C) 102(E)
(D) 102(g)
(E) All the above sections have that requirement.

A

(B) is correct. Pre-AIA 102(B) is a statutory bar, which asks not whether the patent applicant was the first to invent, but rather whether the applicant for patent waited to long to apply for a patent in the United States. Pre-AIA 102(B) specifically says: “A person shall be entitled to a patent unless - the invention was patented or described in a printed publication in this or a foreign country or in public use or on sale in this country, more than one year prior to the date of the application for patent in the United States …. “ As you can see, pre-AIA 102(B) relates to the on-sale bar and public use, among other things. By its express terms, it is not limited to actions by another. Indeed the inventor can create their own problems.

(A) is incorrect because pre-AIA 102(A) is a novelty section, which asks whether certain activities or events happened prior to the date of invention by the applicant for patent. Thus, pre-AIA 102(A) is limited to actions by another. Pre-AIA 102(A) specifically says: “A person shall be entitled to a patent unless - the invention was known or used by others in this country, or patented or described in a printed publication in this or a foreign country, before the invention thereof by the applicant for patent …. “

(C) is incorrect because pre-AIA 102(E) is a novelty section, which asks whether certain activities or events happened prior to the date of invention by the applicant for patent. Pre-AIA

(E) specifically says: “the invention was described in - (I) an application for patent. .. by another filed in the United States before the invention by the applicant for patent or (2) a patent granted on an application for patent by another filed in the United States before the invention by the applicant for patent. … “ The trigger date is “before the invention by the applicant.” Pre- AIA 102(e) is limited to actions “by another.”

(D) is incorrect because pre-AIA 102(g)(I) is the section that determines which applicant of identical claims will receive the patent under first-toinvent principles. Pre-AIA 102(g)(1) specifically says: “A person shall be entitled to a patent unless - during the course of an interference conducted … another inventor involved therein establishes … that before such person’s invention thereof the invention was made by such other inventor and not abandoned, suppressed, or concealed …. “ Pre-AIA 102(g)(2) relates to a situation where the invention was made in the U.S. prior to the date of invention by the applicant for patent. Pre-AIA 102(g)(2) specifically says: “A person is entitled to a patent unless - before such person’s invention thereof, the invention was made in this country by another inventor who had not abandoned, suppressed, or concealed … “ Thus, pre-AIA 102(g)(2) is a novelty section.

(E) is incorrect because (A) is correct.

48
Q

Which of the following sections of pre-AIA 102 cannot be “sworn back” of:
(A) 102(A)
(B) 102(B)
(C) 102(E)
(D) None
(E) 102(A) and 102(E).

A

(B) is correct. Pursuant to 37 C.F.R. 1.l31, when any claim of an application or a patent under reexamination is rejected, an appropriate oath or declaration may be filed to establish invention of the subject matter of the rejected claim prior to the effective date of the reference or activity on which the rejection is based. By filing a 131 affidavit, the applicant is swearing behind a reference to establish an earlier date of invention. Pre-AIA 102(B) is a statutory bar section, which asks not whether the patent applicant was the first to invent, but rather whether the applicant for patent waited too long to apply for a patent in the United States. Thus, swearing behind a 102(B) rejection is impossible because the 131 affidavit seeks to establish an earlier date of invention, which is irrelevant in a 102(B) rejection scenario.

(A) and (C) are incorrect because both pre-AIA 102(A) and pre-AIA 102(e) are novelty sections, which ask whether certain activities or events happened prior to the date of invention by the applicant for patent. Thus, it would be relevant and useful to file a 131 affidavit to establish that the applicant or patent owner in reexamination has an earlier date of invention than the asserted reference.

(D) is incorrect because (B) is correct.

(E) is incorrect because both pre-AIA 102(A) and pre-AIA 102(E) are novelty sections. Thus, rejections under pre-AIA 102(A) and/or pre-AIA 102(E) art can be successfully sworn behind with a 131 affidavit.

49
Q

Which of the following sections of pre-AIA 102 include prior patents and publications anywhere in the world?

(A) 102(A)
(B) 102(B)
(C) 102(E)
(D) 102(g)
(E) 102(A) and 102(B).

A

(E) is correct. Pre-AIA 102(A) specifically says: “A person shall be entitled to a patent unless - the invention was known or used by others in this country, or patented or described in a printed publication in this or a foreign country …. “ Pre-AIA 102(B) specifically says: “A person shall be entitled to a patent unless - the invention was patented or described in a printed publication in this or a foreign country …. “ Thus, both pre-AIA 102(A) and pre-AIA 102(B) pertain to prior patents and publications anywhere in the world.

(A) is incorrect because pre-AIA 102(B) also pertains to prior patents and publications anywhere in the world.

(B) is incorrect because pre-AIA 102(A) also pertains to prior patents and publications anywhere in the world.

(C) is incorrect because pre-AIA 102(E) pertains to U.S. filings or certain applications filed pursuant to the Patent Cooperation Treaty (i.e., applications that designate the U.S. and publish in English). Furthermore, foreign patent applications do not receive a pre-AIA 102(E) date as of their foreign filing date.

(D) is incorrect because pre-AIA 102(g)(1) is the section that determines which applicant of identical claims will receive the patent under first-to-invent principles. Pre-AIA 102(g)(2) relates to a situation where the invention was made in the U.S. prior to the date of invention by the applicant for patent. Neither pre-AIA 102(g)(1) nor (g)(2) relate to prior patents and publications anywhere in the world.

50
Q

Which of the following sections of pre-AIA 102 include an “offer for sale or a public use” in the United States?

(A) 102(A)
(B) 102(B)
(C) 102(E)
(D) 102(g)
(E) 102(A) and 102(B).

A

(B) is correct. Pre-AIA 102(B) is a statutory bar section, which asks not whether the patent applicant was the first to invent, but rather whether the applicant for patent waited too long to apply for a patent in the United States. Pre-AIA 102(B) specifically says: “A person shall be entitled to a patent unless - the invention was patented or described in a printed publication in this or a foreign country or in public use or on sale in this country, more than one year prior to the date of the application for patent in the United States …. “ As you can see, pre-AIA 102(B) relates to the on-sale bar and public use, among other things.

(A) is incorrect because pre-AIA 102(A) is a novelty section, which asks whether certain activities or events happened prior to the date of invention by the applicant for patent. It does not mention offers for sale or public use. Pre-AIA 102(A) specifically says: “A person shall be entitled to a patent unless - the invention was known or used by others in this country, or patented or described in a printed publication in this or a foreign country, before the invention thereof by the applicant for patent. … “

(C) is incorrect because pre-AIA 102(E) pertains to U.S. filings or certain applications filed pursuant to the Patent Cooperation Treaty (i.e., applications that designate the U.S. and publish in English). It does not mention offers for sale or public use.

(D) is incorrect because pre-AIA 102(g)(1) is the section responsible for determining which applicant of identical claims will receive the patent under first to invent principles. Pre-AIA 102(g)(2) relates to a situation where the invention was made in the U.S. prior to the date of invention by the applicant for patent. Neither pre-AIA 102(g)(1) nor (g)(2) relate to offers for sale or public use.

(E) is incorrect because pre-AIA 102(a) is incorrect.

51
Q

Which of the following sections of pre-AIA 102 include “known or used” in the United States?

(A) 102(A)
(B) 102(B)
(C) 102(E)
(D) 102(g)
(E) 102(A) and 102(B).

A

(A) is correct because pre-AIA 102(A) specifically says: “A person shall be entitled to a patent unless - the invention was known or used by others in this country, or patented or described in a printed publication in this or a foreign country, before the invention thereof by the applicant for patent …. “ It is the only section of pre-AIA 102 that uses the phrase “known or used.”

(B) is incorrect because pre-AIA 102(B) does not use the phrase “known or used.” Pre-AIA 102(B) specifically says: “A person shall be entitled to a patent unless - the invention was patented or described in a printed publication in this or a foreign country or in public use or on sale in this country, more than one year prior to the date of the application for patent in the United States …. “ As you can see, pre-AIA 102(B) relates to the on-sale bar and public use, among other things.

(C) is incorrect because pre-AIA 102

(E) does not use the phrase “known or used.” Pre-AIA 102(E) pertains to U.s. filings or certain applications filed pursuant to the Patent Cooperation Treaty (i.e., applications that designate the U.S. and publish in English).

(D) is incorrect because pre-AIA 102(g) does not use the phrase “known or used.” Pre-AIA 102(g)(1) is the section responsible for determining which applicant of identical claims will receive the patent under first-toinvent principles. Pre-AIA 102(g)(2) relates to a situation where the invention was made in the U.S. prior to the date of invention by the applicant for patent.

(E) is incorrect because pre-AIA 102(B) does not use the phrase “known or used.”